Difference between revisions of "2017 AMC 12A Problems/Problem 9"

m (Solution)
m (Added a sidenote)
(5 intermediate revisions by 3 users not shown)
Line 1: Line 1:
==Problem==
+
== Problem ==
  
 
Let <math>S</math> be the set of points <math>(x,y)</math> in the coordinate plane such that two of the three quantities <math>3</math>, <math>x+2</math>, and <math>y-4</math> are equal and the third of the three quantities is no greater than the common value. Which of the following is a correct description of <math>S</math>?
 
Let <math>S</math> be the set of points <math>(x,y)</math> in the coordinate plane such that two of the three quantities <math>3</math>, <math>x+2</math>, and <math>y-4</math> are equal and the third of the three quantities is no greater than the common value. Which of the following is a correct description of <math>S</math>?
Line 5: Line 5:
 
<math> \textbf{(A)}\ \text{a single point} \qquad\textbf{(B)}\ \text{two intersecting lines} \\ \qquad\textbf{(C)}\ \text{three lines whose pairwise intersections are three distinct points} \\ \qquad\textbf{(D)}\ \text{a triangle}\qquad\textbf{(E)}\ \text{three rays with a common point} </math>
 
<math> \textbf{(A)}\ \text{a single point} \qquad\textbf{(B)}\ \text{two intersecting lines} \\ \qquad\textbf{(C)}\ \text{three lines whose pairwise intersections are three distinct points} \\ \qquad\textbf{(D)}\ \text{a triangle}\qquad\textbf{(E)}\ \text{three rays with a common point} </math>
  
==Solution==
+
== Solution ==
If the two equal values are <math>3</math> and <math>x+2</math>, then <math>x=1</math>. Also, <math>y-4\leqslant 3</math> because 3 is the common value. Solving for <math>y</math>, we get <math>y<7</math>. Therefore the portion of the line <math>x=1</math> where <math>y<7</math> is part of <math>S</math>. This is a ray with an endpoint of <math>(1, 7)</math>.
+
If the two equal values are <math>3</math> and <math>x+2</math>, then <math>x=1</math>. Also, <math>y-4\leqslant 3</math> because <math>3</math> is the common value. Solving for <math>y</math>, we get <math>y\leqslant 7</math>. Therefore the portion of the line <math>x=1</math> where <math>y\leqslant 7</math> is part of <math>S</math>. This is a ray with an endpoint of <math>(1, 7)</math>.  
  
Similar to the process above, we assume that the two equal values are <math>3</math> and <math>y-4</math>. Solving the equation <math>3=y-4</math> then <math>y=7</math>. Also, <math>x+2\leqslant 3</math> because 3 is the common value. Solving for <math>x</math>, we get <math>x<1</math>. Therefore the portion of the line <math>y=7</math> where <math>x<1</math> is also part of <math>S</math>. This is another ray with the same endpoint as the above ray: <math>(1, 7)</math>.
+
Similar to the process above, we assume that the two equal values are <math>3</math> and <math>y-4</math>. Solving the equation <math>3=y-4</math> then <math>y=7</math>. Also, <math>x+2\leqslant 3</math> because <math>3</math> is the common value. Solving for <math>x</math>, we get <math>x\leqslant 1</math>. Therefore the portion of the line <math>y=7</math> where <math>x\leqslant 1</math> is also part of <math>S</math>. This is another ray with the same endpoint as the above ray: <math>(1, 7)</math>. (Note that the only answer choice which has rays in it is answer choice <math>E</math>.)
  
If <math>x+2</math> and <math>y-4</math> are the two equal values, then <math>x+2=y-4</math>. Solving the equation for <math>y</math>, we get <math>y=x+6</math>. Also <math>3\leqslant y-4</math> because <math>y-4</math> is one way to express the common value (<math>Using x-2 as the common value works as well). Solving for </math>y<math>, we get </math>y>7<math>. Therefore the portion of the line </math>y=x+6<math> where </math>y>7<math> is part of </math>S<math> like the other two rays. The lowest possible value that can be achieved is also </math>(1, 7)<math>.
+
If <math>x+2</math> and <math>y-4</math> are the two equal values, then <math>x+2=y-4</math>. Solving the equation for <math>y</math>, we get <math>y=x+6</math>. Also <math>3\leqslant y-4</math> because <math>y-4</math> is one way to express the common value (using <math>x-2</math> as the common value works as well). Solving for <math>y</math>, we get <math>y\geqslant 7</math>. Therefore the portion of the line <math>y=x+6</math> where <math>y\geqslant 7</math> is part of <math>S</math> like the other two rays. The lowest possible value that can be achieved is also <math>(1, 7)</math>.
  
Since </math>S<math> is made up of three rays with common endpoint </math>(1, 7)<math>, the answer is </math>\boxed{E}$
+
Since <math>S</math> is made up of three rays with common endpoint <math>(1, 7)</math>, the answer is <math>\boxed{E}</math>.
  
 
Solution by TheMathematicsTiger7
 
Solution by TheMathematicsTiger7
  
==See Also==
+
== See Also ==
 
{{AMC10 box|year=2017|ab=A|num-b=11|num-a=13}}
 
{{AMC10 box|year=2017|ab=A|num-b=11|num-a=13}}
 
{{AMC12 box|year=2017|ab=A|num-b=8|num-a=10}}
 
{{AMC12 box|year=2017|ab=A|num-b=8|num-a=10}}
 
{{MAA Notice}}
 
{{MAA Notice}}

Revision as of 14:09, 19 December 2020

Problem

Let $S$ be the set of points $(x,y)$ in the coordinate plane such that two of the three quantities $3$, $x+2$, and $y-4$ are equal and the third of the three quantities is no greater than the common value. Which of the following is a correct description of $S$?

$\textbf{(A)}\ \text{a single point} \qquad\textbf{(B)}\ \text{two intersecting lines} \\ \qquad\textbf{(C)}\ \text{three lines whose pairwise intersections are three distinct points} \\ \qquad\textbf{(D)}\ \text{a triangle}\qquad\textbf{(E)}\ \text{three rays with a common point}$

Solution

If the two equal values are $3$ and $x+2$, then $x=1$. Also, $y-4\leqslant 3$ because $3$ is the common value. Solving for $y$, we get $y\leqslant 7$. Therefore the portion of the line $x=1$ where $y\leqslant 7$ is part of $S$. This is a ray with an endpoint of $(1, 7)$.

Similar to the process above, we assume that the two equal values are $3$ and $y-4$. Solving the equation $3=y-4$ then $y=7$. Also, $x+2\leqslant 3$ because $3$ is the common value. Solving for $x$, we get $x\leqslant 1$. Therefore the portion of the line $y=7$ where $x\leqslant 1$ is also part of $S$. This is another ray with the same endpoint as the above ray: $(1, 7)$. (Note that the only answer choice which has rays in it is answer choice $E$.)

If $x+2$ and $y-4$ are the two equal values, then $x+2=y-4$. Solving the equation for $y$, we get $y=x+6$. Also $3\leqslant y-4$ because $y-4$ is one way to express the common value (using $x-2$ as the common value works as well). Solving for $y$, we get $y\geqslant 7$. Therefore the portion of the line $y=x+6$ where $y\geqslant 7$ is part of $S$ like the other two rays. The lowest possible value that can be achieved is also $(1, 7)$.

Since $S$ is made up of three rays with common endpoint $(1, 7)$, the answer is $\boxed{E}$.

Solution by TheMathematicsTiger7

See Also

2017 AMC 10A (ProblemsAnswer KeyResources)
Preceded by
Problem 11
Followed by
Problem 13
1 2 3 4 5 6 7 8 9 10 11 12 13 14 15 16 17 18 19 20 21 22 23 24 25
All AMC 10 Problems and Solutions
2017 AMC 12A (ProblemsAnswer KeyResources)
Preceded by
Problem 8
Followed by
Problem 10
1 2 3 4 5 6 7 8 9 10 11 12 13 14 15 16 17 18 19 20 21 22 23 24 25
All AMC 12 Problems and Solutions

The problems on this page are copyrighted by the Mathematical Association of America's American Mathematics Competitions. AMC logo.png